Integral of a complex exponential

Click For Summary
The integral of a complex exponential involving a Hermitian matrix is discussed, specifically the expression ∫_V dV e^(iA_{j,k}x^{j}x^{k}) = δ(DetA)(2π)^{n}. It is noted that the integral diverges when the matrix A is not invertible, indicated by DetA = 0. In such cases, at least one eigenvalue of A is zero, causing the exponential to equal one and leading to divergence. The discussion suggests working in a diagonal basis for matrix A to better analyze the situation. Understanding the behavior of the integral in relation to the determinant is crucial for resolving the divergence issue.
mhill
Messages
180
Reaction score
1

Homework Statement



let be A_{i,j} a Hermitian Matrix with only real values then

\int_{V} dV e^{iA_{j,k}x^{j}x^{k}}= \delta (DetA) (2\pi)^{n} (1)

Homework Equations



\int_{V} dV e^{iA_{j,k}x^{j}x^{k}} = \delta (DetA) (2\pi)^{n}

The Attempt at a Solution



the idea is that the integral (1) is divergent when the Matrix A is not invertible Det=A and the Dirac delta is not defined at x=0 ,for example if detA=0 then at least one of the eigenvalues is 0 so the exponential takes the value 1 and the integral is divergent
 
Physics news on Phys.org
Try working in the basis in which A is diagonal.
 
Question: A clock's minute hand has length 4 and its hour hand has length 3. What is the distance between the tips at the moment when it is increasing most rapidly?(Putnam Exam Question) Answer: Making assumption that both the hands moves at constant angular velocities, the answer is ## \sqrt{7} .## But don't you think this assumption is somewhat doubtful and wrong?

Similar threads

  • · Replies 14 ·
Replies
14
Views
2K
  • · Replies 5 ·
Replies
5
Views
2K
  • · Replies 5 ·
Replies
5
Views
2K
Replies
1
Views
1K
  • · Replies 2 ·
Replies
2
Views
2K
Replies
2
Views
2K
  • · Replies 6 ·
Replies
6
Views
2K
Replies
1
Views
2K
Replies
8
Views
2K